OBM 2015 - Nível 3 - P4

Problema 4

Seja n um inteiro positivo e sejam n=d_1 data-recalc-dims=d_2>\dots>d_k=1" /> seus divisores positivos.

a) Prove que

d_1-d_2+d_3-\dots+(-1)^{k-1}d_k=n-1

se, e somente se n é primo ou n=4.

b) Determine os três inteiros positivos n para os quais

d_1-d_2+d_3-\dots+(-1)^{k-1}d_k=n-4.

Solução de João Rafael:

a) Temos que:

S=n-d_2+\dots+(-1)^{k-1}d_k=n-1

\Rightarrow(-d_2+d_3)+(-d_4+d_5)+\dots+(-1)^{k-1}d_k=-1

Porém como d_i data-recalc-dims=d_{i+1}\Rightarrow d_i-d_{i+1}\geq1" />:

-1=(-d_2+d_3)+\dots+(-1)^{k-1}d_k\leq-1\lfloor\frac{k}{2}\rfloor com k\geq1

\Rightarrow k=2 ou k=3

Se k=2 então os divisores de n são n e 1, logo n é primo.

Se k=3 então:

-d_2+d_3=-1\Rightarrow d_2-1=1\Rightarrow d_2=2

Como d_2 é o divisor do meio de n temos que:

d_2=2=\sqrt n\Rightarrow n=4

Basta fazer as contas e checar que de fato para n=4 e n primo vale que S=n-1

b) Temos que:

S=n-d_2+\dots+(-1)^{k-1}d_k=n-4\Rightarrow(-d_2+d_3)+(-d_4+d_5)+\dots+(-1)^{k-1}=-4

Como vimos (-d_{i}+d_{i+1})\leq-1 e isso nos da que k\leq9.

Certamente k\neq\{1,2\}(k=1 é impossível; k=2 chegamos em n primo e S=n-1\neq n-4)

Se k=3:

n-d_2+1=n-4\Rightarrow d_2=5=\sqrt{n}\Rightarrow n=25

Fazendo as contas para n=25 checamos que de fato n=25 é solução.

Se k=4:

-d_2+d_3=-3\Rightarrow d_2-d_3=3

Note que se d_{k-1} sempre deverá ser primo pois se caso ele seja composto os seus divisores dividiriam n e seriam menores que ele o que é um absurdo pois d_{k-1} é o menor divisor de n maior que 1. Assim d_3 é primo e isso implica que ou d_2 é potência de d_3 ou d_2 é primo pois caso o contrário existiria outro divisor de n além de d_2 e d_3 pois se d_2 for composto e não for potência de d_3 teriamos:

d_2=ab com mdc(b,d_3)=1 e a,b\ne1\Rightarrow b|n e é diferente de d_2, d_3, 1 e n, absurdo.

Logo se d_2 for potência de d_3 então d_2=d_3^2\Rightarrow d_3^2-d_3=3\Rightarrow d_2\notin \mathbb{R}

Se d_2 for primo temos que d_2 e d_3 tem paridade diferente e são primos com d_2=d_3+3\Rightarrow d_3=2 e d_2=5\Rightarrow n=d_2d_3=10

Fazendo as contas com n=10 checamos que é uma solução.

Se k=5:

-d_2+d_3-d_4=-5\Rightarrow d_2+d_4-d_3=5

Note que sendo n=P_1^{\alpha_1}P_2^{\alpha_2}\dots P_k^{\alpha_k} temos que:

nº de divisores de n=\sigma_0(n)=(\alpha_1+1)(\alpha_2+1)\dots(\alpha_k+1)=5

Como 5 é primo então \alpha_1=4 e os demais expoentes serão 0. Assim n será potência de primo e logo:

d_4=P_1, d_3=P_1^2 e d_2=P_1^3\Rightarrow P_1^3+P_1-P_1^2=5\Rightarrow P_1(P_1^2+1-P_1)=5\Rightarrow P_1=5 pois 5 é primo e logo P_1^2+1-P_1=21\neq1, absurdo.

Logo nesse caso não há solução.

Se k=6:

-d_2+d_3-d_4+d_5=-3\Rightarrow -3=(d_3-d_2)+(d_5-d_4)\leq-2

A igualdade ocorre com d_3=d_2+1 e d_5=d_4+1. Porém sabemos que:

d_2d_5=d_3d_4\Rightarrow d_2(d_4+1)=(d_2+1)d_4\Rightarrow d_2=d_4, absurdo.

Temos então que:

-d_2+d_3-d_4+d_5=-3\Rightarrow -d_2+d_3=-2 ou -d_4+d_5=-2

Se d_3-d_2=-2 e d_5-d_4=-1 então:

d_2=d_3+2 e d_4=d_5+1\Rightarrow d_2d_5=d_3d_4\Rightarrow (d_3+2)d_5=d_3(d_5+1)\Rightarrow 2d_5=d_3

\Rightarrow como d_2=d_3+2\Rightarrow d_2=2d_5+2=2d_4

Assim os divisores próprios de n são 2d_5+2, 2d_5, d_5+1 e d_5. Porém note que d_4=d_5+1 data-recalc-dims=d_5" /> e d_5+1|n\Rightarrow d_5+1=d_5^2 ou d_5+1 é primo.

Se d_5+1=d_5^2\Rightarrow d_5\notin\mathbb{R}

Se d_5+1 é primo, como d_5 também é primo, então d_5=2\Rightarrow n=(2d_5+2)d_5=6\times 2=12 o que se fizermos as contas de fato é solução.

Se d_3-d_2=-1 e d_5-d_4=-2:

d_2=d_3+1 e d_4=d_5+1\Rightarrow(d_3+1)d_5=d_3(d_5+2)\Rightarrow d_5=2d_3\Rightarrow d_5 data-recalc-dims=d_3" /> , absurdo.

Assim para k=6 temos n=12.

Se k=7:

\sigma_0(n)=(\alpha_1+1)\dots(\alpha_k+1)=7\Rightarrow \alpha_1=6 e os demais \alpha_i=0\Rightarrow d_i=P_1^{7-i}

\Rightarrow -P_1^5+P_1^4-P_1^3+P_1^2-P_1=-5\Rightarrow P_1(-P_1^4+P_1^3-P_1^2+P_1-1)=-5\Rightarrow P_1=5\Rightarrow -P_1^4+P_1^3-P_1^2+P_1-1=-1 o que certamente é falso.

Logo nesse caso não há soluções.

Se k=8:

-3\geq-d_2+d_3-d_4+d_5-d_6+d_7=-3

A inequação tem que a igualdade quando -d_i+d_{i+1}=1 para i=\{2, 4, 6\}

\Rightarrow d_3=d_2+1 e d_7=d_6+1.

Como d_2d_7=d_3d_6:

d_2(d_6+1)=(d_2+1)d_6\Rightarrow d_2=d_6, absurdo e logo nesse caso não há soluções.

Se k=9:

(-d_2+d_3)+(-d_4+d_5)+(-d_6+d_7)-d_8=-5

Como d_8\geq2\Rightarrow -d_8\leq-2:

(-d_2+d_3)+(-d_4+d_5)+(-d_6+d_7) data-recalc-dims=(-d_2+d_3)+(-d_4+d_5)+(-d_6+d_7)-2\geq(-d_2+d_3)+(-d_4+d_5)+(-d_6+d_7)-d_8=-5" />

\Rightarrow(-d_2+d_3)+(-d_4+d_5)+(-d_6+d_7) data-recalc-dims=-3" />

Porém como -d_i+d_{i+1}\leq-1

-3 data-recalc-dims=(-d_2+d_3)+(-d_4+d_5)+(-d_6+d_7)>-3" />, absurdo.

Assim, temos que todos os n que satisfazem o enunciado são n=\{10, 12, 25\}.\blacksquare

Resposta: 10, 12, 25.